Diễn Đàn MathScopeDiễn Đàn MathScope
  Diễn Đàn MathScope
Ghi Danh Hỏi/Ðáp Thành Viên Social Groups Lịch Ðánh Dấu Ðã Ðọc

Go Back   Diễn Đàn MathScope > Sơ Cấp > Việt Nam và IMO > 2015

News & Announcements

Ngoài một số quy định đã được nêu trong phần Quy định của Ghi Danh , mọi người tranh thủ bỏ ra 5 phút để đọc thêm một số Quy định sau để khỏi bị treo nick ở MathScope nhé !

* Nội quy MathScope.Org

* Một số quy định chung !

* Quy định về việc viết bài trong diễn đàn MathScope

* Nếu bạn muốn gia nhập đội ngũ BQT thì vui lòng tham gia tại đây

* Những câu hỏi thường gặp

* Về việc viết bài trong Box Đại học và Sau đại học


Trả lời Gởi Ðề Tài Mới
 
Ðiều Chỉnh Xếp Bài
Old 26-03-2015, 10:56 AM   #16
huynhcongbang
Administrator

 
huynhcongbang's Avatar
 
Tham gia ngày: Feb 2009
Đến từ: Ho Chi Minh City
Bài gởi: 2,413
Thanks: 2,165
Thanked 4,188 Times in 1,381 Posts
Gửi tin nhắn qua Yahoo chát tới huynhcongbang
Gửi mọi người đề ngày 1 mình đã gõ Latex xong.

Chờ đề ngày 2 thôi...
[RIGHT][I][B]Nguồn: MathScope.ORG[/B][/I][/RIGHT]
 
File Kèm Theo
Kiểu File : pdf VNTST 2015 - Day 1.pdf (106.6 KB, 367 lần tải)
__________________
Sự im lặng của bầy mèo
huynhcongbang is offline   Trả Lời Với Trích Dẫn
The Following 6 Users Say Thank You to huynhcongbang For This Useful Post:
dangvip123tb (26-03-2015), n.t.tuan (26-03-2015), n.v.thanh (27-03-2015), quocbaoct10 (26-03-2015), thaygiaocht (26-03-2015), thiendieu96 (27-03-2015)
Old 26-03-2015, 01:34 PM   #17
n.t.tuan
+Thành Viên+
 
n.t.tuan's Avatar
 
Tham gia ngày: Nov 2007
Bài gởi: 1,250
Thanks: 119
Thanked 616 Times in 249 Posts
Đề ngày 2 đâu các bạn ơi? Thấy bảo bài Đại số khó lắm.
[RIGHT][I][B]Nguồn: MathScope.ORG[/B][/I][/RIGHT]
 
__________________
T.
n.t.tuan is offline   Trả Lời Với Trích Dẫn
The Following User Says Thank You to n.t.tuan For This Useful Post:
dangvip123tb (26-03-2015)
Old 26-03-2015, 02:24 PM   #18
huynhcongbang
Administrator

 
huynhcongbang's Avatar
 
Tham gia ngày: Feb 2009
Đến từ: Ho Chi Minh City
Bài gởi: 2,413
Thanks: 2,165
Thanked 4,188 Times in 1,381 Posts
Gửi tin nhắn qua Yahoo chát tới huynhcongbang
Dưới đây là đề thi ngày 2, mình xin được từ bạn Lê Nhật Hoàng (LNH).

Bài 4.

Trong một kỳ thi vấn đáp, có $100$ thí sinh và $25$ vị giám khảo, mỗi thí sinh thích ít nhất $10$ giám khảo.

a) Chứng minh rằng có thể chọn ra $7$ giám khảo mà mỗi thí sinh đều thích ít nhất $1$ trong $7$ người đó.
b) Chứng minh rằng có thể sắp xếp lịch thi sao cho mỗi thí sinh được đúng $1$ giám khảo mình thích hỏi và mỗi giám khảo hỏi không quá $10$ thí sinh.

Bài 5.

Cho tam giác$ABC$ nhọn và có điểm $P$ nằm trong tam giác sao cho $\angle APB=\angle APC = \alpha$ và $\alpha>180{}^\circ - \angle BAC $. Đường tròn ngoại tiếp tam giác $APB$ cắt $AC$ ở $E,$ đường tròn ngoại tiếp tam giác $APC$ cắt $AB$ ở $F$ . Gọi $Q$ là điểm nằm trong tam giác $AEF$ sao cho $\angle AQE=\angle AQF$. Gọi $D$ là điểm đối xứng với $Q$ qua $EF$ , phân giác góc $EDF$ cắt $AP$ tại $T.$

a) Chứng minh rằng $\angle DET=\angle ABC,\angle DFT=\angle ACB$ .
b) Đường thẳng $PA$ cắt các đường thẳng $DE,DF$ lần lượt tại $M,N$ . Gọi $I,J$ lần lượt là tâm đường tròn nội tiếp các tam giác $PEM,PFN$ và $K$ là tâm đường tròn ngoại tiếp tam giác $DIJ$ . Đường thẳng $DT$ cắt $(K)$ tại $H$. Chứng minh rằng $HK$ đi qua tâm đường tròn nội tiếp của tam giác $DMN.$

Bài 6.

Tìm số nguyên dương $n$ nhỏ nhất sao cho tồn tại $n$ số thực thỏa mãn đồng thời các điều kiện:

i) Tổng của $n$ số đó dương.
ii) Tổng lập phương của $n$ số đó âm.
iii) Tổng lũy thừa bậc $5$ của $n$ số đó dương.

----------------------

Dưới đây là file Latex cho ngày 2.
[RIGHT][I][B]Nguồn: MathScope.ORG[/B][/I][/RIGHT]
 
File Kèm Theo
Kiểu File : pdf VNTST 2015 - Day 2.pdf (102.1 KB, 299 lần tải)
__________________
Sự im lặng của bầy mèo

thay đổi nội dung bởi: huynhcongbang, 26-03-2015 lúc 02:46 PM
huynhcongbang is offline   Trả Lời Với Trích Dẫn
The Following 10 Users Say Thank You to huynhcongbang For This Useful Post:
dangvip123tb (26-03-2015), Mộc Gia San (26-03-2015), n.t.tuan (26-03-2015), n.v.thanh (27-03-2015), quocbaoct10 (26-03-2015), thanhgand (28-03-2015), thaygiaocht (26-03-2015), thiendieu96 (27-03-2015), vantienducdh (26-03-2015), whatever2507 (26-03-2015)
Old 26-03-2015, 03:43 PM   #19
thaygiaocht
+Thành Viên+
 
thaygiaocht's Avatar
 
Tham gia ngày: Aug 2012
Đến từ: Chuyên Hà Tĩnh
Bài gởi: 165
Thanks: 793
Thanked 216 Times in 93 Posts
Cảm nhận đầu tiên là đề năm nay có hình thức phát biểu bài toán đẹp hơn và có mức độ khó hơn mọi năm.
[RIGHT][I][B]Nguồn: MathScope.ORG[/B][/I][/RIGHT]
 
__________________
https://www.facebook.com/thaygiaocht
thaygiaocht is offline   Trả Lời Với Trích Dẫn
The Following 3 Users Say Thank You to thaygiaocht For This Useful Post:
dangvip123tb (26-03-2015), thiendieu96 (27-03-2015), vantienducdh (26-03-2015)
Old 26-03-2015, 04:18 PM   #20
Nvthe_cht.
+Thành Viên+
 
Tham gia ngày: Nov 2013
Bài gởi: 69
Thanks: 15
Thanked 36 Times in 24 Posts

Đây là lời giải bài hình ngày 1 của mình.
Gọi $T$ là giao điểm của $BC$ và $EF$. $N'$ là giao điểm của $TH$ với $AI$, $M'$ là điểm đối xứng của $N'$ qua $BC$.
Theo các kết quả quen thuộc thì ta có $T,K,A$ thẳng hàng. $TH$ vuông góc $AI$.
Ta có: $\widehat{M'DI}=\widehat{N'DI}=\widehat{TAN}=\wide hat{TDK}$
Nên $K, D, M$ thẳng hàng.
Mặt khác không khó để có $BHN'C$ nội tiếp nên $M'$ thuộc đường tròn $(O)$
Từ các điều trên ta có $M'$ trùng $M$ hay $N'$ trùng $N$, nói cách khác $N$ nằm trên đường tròn $(BHC)$ cố định.
Chú ý: $(BHC)$ cố định vì $\widehat{BHC}$ không đổi và $BC$ cố định.

b. Gọi $S$ là giao điểm của $PQ$ với $AT$. Khi đó $A$ nằm trên đường đối cực của $S$ với đường tròn $(APQ)$, mặt khác chú ý rằng $A(SD,PQ)=-1$ nên $AD$ là đường đối cực của $S$ đối với $(APQ)$, hay $PP,QQ,AD$ đồng quy, từ đây thì ta có $AJ,AD$ đẳng giác trong góc $\widehat{PAQ}$, do vậy $AJ$ đi qua tâm O cố định.
[RIGHT][I][B]Nguồn: MathScope.ORG[/B][/I][/RIGHT]
 
Nvthe_cht. is offline   Trả Lời Với Trích Dẫn
The Following 2 Users Say Thank You to Nvthe_cht. For This Useful Post:
dangvip123tb (26-03-2015), sieusieu90 (26-03-2015)
Old 26-03-2015, 05:21 PM   #21
huynhcongbang
Administrator

 
huynhcongbang's Avatar
 
Tham gia ngày: Feb 2009
Đến từ: Ho Chi Minh City
Bài gởi: 2,413
Thanks: 2,165
Thanked 4,188 Times in 1,381 Posts
Gửi tin nhắn qua Yahoo chát tới huynhcongbang
Bài cuối thì mình mới có một số ý thế này:

Dễ thấy $n=1$ là không thỏa.

- Với $n=2$, giả sử ta có 2 số $a,b$ thì phải có hệ điều kiện $$\left\{\begin{matrix}
a+b>0\\ a^3+b^3<0
\\ a^5+b^5>0
\end{matrix}\right.$$ Dễ thấy điều này cũng không xảy ra được vì $a^3+b^3=(a+b)(a^2-ab+b^2)$ là luôn cùng dấu với $a+b$.

- Với $n=3$, xét 3 số $(a,b,c)$, ta có hệ điều kiện $$\left\{\begin{matrix}
a+b+c>0\\ a^3+b^3+c^3<0
\\ a^5+b^5+c^5>0
\end{matrix}\right.$$ Ta sẽ chứng minh điều này là không thể xảy ra.

Rõ ràng trong 3 số phải có ít nhất 1 số âm và ít nhất 1 số dương. Ta xét 2 trường hợp:

+ Nếu có 2 số âm, 1 số dương, giả sử là $a,b<0, c > 0$ thì đặt $a'= -a>0, b' = -b$, ta có hệ mới $$\left\{\begin{matrix}
a'+b'<c\\ a'^3+b'^3>c^3
\\ a'^5+b'^5<c^5
\end{matrix}\right.$$ Nhân BĐT thứ nhất và thứ hai, ta có $(a'+b')(a'^5+b'^5)<c'^6 = (c'^3)^2 < (a'^3+b'^3)^2$, sai theo BĐT Cauchy-Schwarz.

+ Nếu có 2 số dương, 1 số âm, giả sử là $a,b>0, c < 0$ thì đặt $c'= -c>0$, ta có hệ mới $$\left\{\begin{matrix}
a+b>c'\\ a^3+b^3<c'^3
\\ a^5+b^5>c'^5
\end{matrix}\right.$$ Đến đây ta cũng chứng minh được BĐT này sai.

- Với $n=5$, ta chỉ ra được một bộ thỏa mãn là $$(a,b,c,d,e) = (-7,-7,2,5,8).$$

Do đó, chỉ còn trường hợp $n=4$ (mình đoán là không thỏa, nhưng chưa chứng minh được).
[RIGHT][I][B]Nguồn: MathScope.ORG[/B][/I][/RIGHT]
 
__________________
Sự im lặng của bầy mèo
huynhcongbang is offline   Trả Lời Với Trích Dẫn
The Following 3 Users Say Thank You to huynhcongbang For This Useful Post:
dangvip123tb (26-03-2015), thiendieu96 (27-03-2015), vantienducdh (26-03-2015)
Old 26-03-2015, 06:43 PM   #22
Traum
Moderator
 
Traum's Avatar
 
Tham gia ngày: Nov 2007
Đến từ: cyber world
Bài gởi: 413
Thanks: 14
Thanked 466 Times in 171 Posts
Bài 6: Chứng minh n = 4 không thỏa.

Ta có $a + b + c + d > 0$, $a^3 + b^3 + c^3 + d^3 < 0$ và $a^5 + b^5 + c^5 + d^5 > 0$. Nên trong 4 số $a$, $b$, $c$, $d$ có ít nhất 1 số âm và 1 số dương.

Xét 3 trường hợp:
TH1: $b > 0$, $c > 0$.
Khi đó ta thay $d$ bởi $-d$ thì điều kiện có thể viết lại thành: $a + b + c > d$, $a^3 + b^3 + c^3 < d^3$, $a^5 + b^5 + c^5 > d^5$.
Gọi $t > 1$ sao cho $t^3(a^3 + b^3 + c^3) = d^3$. Khi đó nếu thay $a$, $b$, $c$ bởi $a_1 = at$, $b_1 = bt$, $c_1 = ct$ thì ta có:
$a_1 + b_ 1 + c_1 = t(a+b+c) > d$, $a_1^3 + b_1^3 + c_1^3 = d^3$ và $a_1^5 + b_1^5 + c_1^5 = t^5(a^5 + b^5 + c^5) > d^5$.
Tuy nhiên khi đó thì vì $a_1, b_1, c_1 < d$ nên $1 = \left(\frac{a_1}{d}\right)^3 + \left(\frac{b_1}{d}\right)^3 + \left(\frac{c_1}{d}\right)^3 > \left(\frac{a_1}{d}\right)^5 + \left(\frac{b_1}{d}\right)^5 + \left(\frac{c_1}{d}\right)^5 > 1$. Vô lý.

TH2: $b < 0$, $c < 0$ tương tự TH1.

TH3: $b > 0$, $c < 0$.
Thay $c$ bởi $-c$, ta viết lại thành $a+b > c+d$, $a^3 + b^3 < c^3 + d^5$, $a^5 + b^5 > c^5 + d^5$ với $a, b, c, d > 0$. Khi đó tương tự như TH1, TH2 ta xét thay $a$, $b$ bởi $at$, $bt$ với $t>1$ sao cho $a^3 + b^3 = c^3 + d^3$.
Giả sử $a \ge b$, $c \ge d$.
TH3a. $a$ là số lớn nhất hay $a > c$. Khi đó $a^3 > c^3 \ge d^3 > b^3$. Ta có hàm số $f(x) = x^{\frac{1}{3}}$ là hàm lõm trên $R^+$ nên $f(a^3) + f(b^3) \le f(c^3) + f(d^3)$ hay $a + b \le c + d$. Vô lý.
TH3b. $c$ là số lớn nhất hay $c > a$. Khi đó $c^3 > a^3 > b^3 > d^3$. Ta có hàm số $f(x) = x^{\frac{5}{3}}$ là hàm lồi trên $R^{+}$ nên $f(a^3) + f(b^3) \le f(c^3) + f(d^3)$ hay $a^5 + b^5 \le c^5 + d^5$. Vô lý.

Vậy ta có $n=4$ không thỏa. Hiển nhiên $n=2,3$ cũng không thỏa. Với $n=5$ thì lấy như post trên $-7, -7, 2, 5, 8$ thỏa mãn.
[RIGHT][I][B]Nguồn: MathScope.ORG[/B][/I][/RIGHT]
 
__________________
Traum is giấc mơ.

thay đổi nội dung bởi: Traum, 26-03-2015 lúc 07:07 PM
Traum is offline   Trả Lời Với Trích Dẫn
The Following 5 Users Say Thank You to Traum For This Useful Post:
dangvip123tb (01-04-2015), huynhcongbang (27-03-2015), quocbaoct10 (26-03-2015), thaygiaocht (26-03-2015), vantienducdh (26-03-2015)
Old 26-03-2015, 06:49 PM   #23
quocbaoct10
+Thành Viên Danh Dự+
 
quocbaoct10's Avatar
 
Tham gia ngày: Oct 2012
Đến từ: THPT chuyên Lê Quý Đôn-Nha Trang-Khánh Hòa
Bài gởi: 539
Thanks: 292
Thanked 365 Times in 217 Posts
Bài 4
a) đặt $A_i$ là tập các giám khảo mà học sinh thứ $i$ thích, ta có $|A_i| \ge 10$.
Khi đấy theo nguyên lí Dirichlet, ta tìm được 1 giám khảo được thích bởi ít nhất $\frac{\sum^{100}_{i=1} |A_i|}{25} =40 $. Bỏ đi 40 học sinh này và vị giám khảo được thích bởi 40 học sinh, ta được số học sinh còn lại là 60 và số giám khảo còn lại là 24. Tiếp tục áp dụng nguyên lí Dirichlet 1 cách liên tục, ta sẽ tìm được 7 vị giáo khảo thỏa ycđb.
b) Ta sẽ tìm cách ghép các học sinh thành nhiều nhất là 25 nhóm và ít nhất là 10 nhóm sao cho mỗi nhóm này thích một giám khảo khác nhau. Ta chỉ cần chứng minh được trường hợp chia thành các nhóm $T$ có ít hơn hoặc bằng 3 người thích 1 giám khảo, khi đấy nếu trong những nhóm có nhiều hơn 11 người thích 1 giám khảo, ta sẽ "chia sẻ" sang nhóm $T$ (vì mỗi người thích ít nhất 10 giám khảo nên việc chia sẽ là hoàn toàn dễ dàng).
Bổ đề: phải có ít nhất 5 nhóm có ít nhất 4 người sao cho mỗi nhóm thích 1 giám khảo phân biệt.
Chứng minh: dễ dàng chứng minh bằng dirichlet
Trở lại bài toán: Vì theo bổ đề, ta có ít nhất 5 nhóm 4 người có thể thích 1 giám khảo phân biệt nên ta sẽ chia thành 5 nhóm 8 người và 20 nhóm 3 người. xét trong các nhóm 8 người, ta có thể chuyển nhiều nhất là 4 người khác sang các nhóm khác (vì 4 người này có thể không thích chung 1 giám khảo như 4 nguời còn lại), mà có thảy 5 nhóm cần chuyển người, nên ta chuyển đi nhiều nhất là 20 người và 20 người này hoàn toàn có thể chuyển đến nhiều nhất là 20 nhóm (mỗi người chuyển đến 1 nhóm). Nếu như tồn tại nhóm $T$ mà học sinh ở nhóm này thích cùng 1 vị giám khảo với 1 trong 5 nhóm "4 người" thì thì ta sẽ ghép nhóm này vào nhóm 4 người trên. Như vậy, ta đã xây dựng được nhiều nhất là 25 nhóm học sinh sao cho mỗi học sinh này thích 1 giám khảo khác nhau .
[RIGHT][I][B]Nguồn: MathScope.ORG[/B][/I][/RIGHT]
 
__________________
i'll try my best.

thay đổi nội dung bởi: quocbaoct10, 26-03-2015 lúc 10:24 PM
quocbaoct10 is offline   Trả Lời Với Trích Dẫn
The Following 3 Users Say Thank You to quocbaoct10 For This Useful Post:
dangvip123tb (01-04-2015), hoang_kkk (01-04-2015), huynhcongbang (27-03-2015)
Old 26-03-2015, 07:03 PM   #24
Traum
Moderator
 
Traum's Avatar
 
Tham gia ngày: Nov 2007
Đến từ: cyber world
Bài gởi: 413
Thanks: 14
Thanked 466 Times in 171 Posts
Trích:
Nguyên văn bởi chemthan View Post
Hình như TH3 sai rồi @, thấy không sử dụng đến $a^5 + b^5 > c^5 + d^5$. Chỗ giả sử $a$ lớn nhất là sai, $a$ chỉ bình đẳng với $b$ thôi!
Làm vội nên quên mất vai trò của của chúng. Nếu a không lớn nhất thì phải sử dụng điều kiện cho lũy thừa 5. Như trên.
[RIGHT][I][B]Nguồn: MathScope.ORG[/B][/I][/RIGHT]
 
__________________
Traum is giấc mơ.
Traum is offline   Trả Lời Với Trích Dẫn
The Following User Says Thank You to Traum For This Useful Post:
thaygiaocht (26-03-2015)
Old 26-03-2015, 10:04 PM   #25
dinhngoctung
+Thành Viên+
 
Tham gia ngày: Mar 2015
Bài gởi: 1
Thanks: 0
Thanked 0 Times in 0 Posts
Bài hình đề gõ thiếu, 4 góc đều bằng alpha.
[RIGHT][I][B]Nguồn: MathScope.ORG[/B][/I][/RIGHT]
 
dinhngoctung is offline   Trả Lời Với Trích Dẫn
Old 26-03-2015, 10:50 PM   #26
Kelacloi
+Thành Viên+
 
Tham gia ngày: Mar 2011
Bài gởi: 252
Thanks: 50
Thanked 164 Times in 114 Posts
Hello mọi người, em xuất hiện trễ
Em bổ sung bài 1b hướng khác.
Đó là chứng minh 2 nhận xét:
Nhận xét 1:
"Nếu bộ $d_0,d_1,..,d_n \ge 0$ thoả:
i) $2015=d_0+d_1\alpha^1+d_2\alpha^2+..+d_n\alpha^n$
i) $d_0+..+d_n$ min
thì $ 0 \le d_k \le 4 \forall 0 \le k \le n$"

Nhận xét 2:
Nếu tồn tại 2 bộ số $(e_0,e_1,..,e_m)$ và $(d_0,d_1,..,d_n)$
thoả:
1) $ 0\le e_k \le 4 ; 0 \le d_k \le 4$
2) $d_0+d_1\alpha^1+d_2\alpha^2+..+d_n\alpha^n=e_0+e_ 1\alpha^1+e_2\alpha^2+..+e_m\alpha^m$
thì 2 bộ đó trùng nhau "
[RIGHT][I][B]Nguồn: MathScope.ORG[/B][/I][/RIGHT]
 
__________________
Kelacloi is offline   Trả Lời Với Trích Dẫn
The Following 3 Users Say Thank You to Kelacloi For This Useful Post:
dangvip123tb (01-04-2015), huynhcongbang (27-03-2015), vantienducdh (26-03-2015)
Old 26-03-2015, 11:53 PM   #27
Nguyen Van Linh
Moderator
 
Tham gia ngày: Aug 2009
Đến từ: Hà Nội
Bài gởi: 277
Thanks: 69
Thanked 323 Times in 145 Posts
Bài 5 vừa lạ vừa quen :3
a) Dễ thấy tứ giác $BEFC$ nội tiếp đồng thời $\angle APB=\angle APC=\angle AQE=\angle AQF=\alpha$ nên $AQ$ và $AP$ đẳng giác trong góc $BAC.$
Gọi $T'$ là điểm liên hợp đẳng giác của $Q$ trong tam giác $AEF$. Hiển nhiên $T'\in AT.$
Gọi $X$ là điểm đối xứng với $T'$ qua $EF$.
Ta có $\angle X'EF=\angle T'EF=\angle AEQ, \angle XFE=\angle T'FE=\angle AFQ$, suy ra $A$ và $X$ liên hợp đẳng giác trong tam giác $EQF$. Mà $QA$ là phân giác $\angle EQF$ nên $X,Q,A$ thẳng hàng.
Do đối xứng của $T'$ qua $EF$ nằm trên $QA$ nên $T'$ nằm trên đối xứng của $QA$ qua $EF$. Mà $D$ đối xứng với $Q$ qua $EF, DT$ là phân giác $\angle EDF$ nên $DT$ đối xứng với $QA$ qua $EF$. Từ đó $T'\equiv T.$
Suy ra $\angle DET=\angle XEQ=\angle FEA=\angle ABC$. Tương tự $\angle DFT=\angle ACB.$
b) Ta có $\angle PED=\angle DEF+\angle FEA+\angle PEA=\angle QEF+\angle ABC+\angle PBA=\angle ABC+\angle PBA+\angle PBC=2\angle BAC$. Theo câu a, $\angle DET=\angle ABC$ nên $ET$ là phân giác $\angle DEA$. Tương tự $FT$ là phân giác $\angle DFA$, suy ra tứ giác $DEPF$ ngoại tiếp đường tròn tâm $T.$
Qua $D$ kẻ tiếp tuyến thứ hai tới $(I)$, cắt $PT$ tại $L.$
Tứ giác $DEPL, DEPF$ ngoại tiếp nên $DF-PF=DE-PE=DL-PL$, suy ra $DLPF$ ngoại tiếp, hay $DL$ là tiếp tuyến chung của $(I)$ và $(J)$. Từ đó $\angle IDJ=\frac{1}{2}\angle EDF.$
Mặt khác, gọi $Y$ là tâm nội tiếp tam giác $DMN$, ta có $\angle IYJ=180^\circ-\angle MIN=90^\circ-\frac{1}{2}\angle MDN=\frac{1}{2}\angle EDF.$
Suy ra $D,Y,I,J$ nội tiếp. Hiển nhiên $\angle YDH=90^\circ$ nên $YH$ là đường kính của $(DIJ)$. Suy ra $YH$ đi qua $K.$
[RIGHT][I][B]Nguồn: MathScope.ORG[/B][/I][/RIGHT]
 
Hình Kèm Theo
Kiểu File : png P5.png (34.0 KB, 79 lần tải)

thay đổi nội dung bởi: Nguyen Van Linh, 27-03-2015 lúc 01:05 AM
Nguyen Van Linh is offline   Trả Lời Với Trích Dẫn
The Following 5 Users Say Thank You to Nguyen Van Linh For This Useful Post:
dangvip123tb (01-04-2015), huynhcongbang (27-03-2015), pco (27-03-2015), thaygiaocht (27-03-2015), thiendieu96 (27-03-2015)
Old 26-03-2015, 11:58 PM   #28
Nguyen Van Linh
Moderator
 
Tham gia ngày: Aug 2009
Đến từ: Hà Nội
Bài gởi: 277
Thanks: 69
Thanked 323 Times in 145 Posts
Bài số 5 là kết hợp của 2 bài toán. Câu a tổng quát từ bài toán đồng quy liên quan đến điểm Fermat, xem tại [Only registered and activated users can see links. ]
Câu b bắt nguồn từ bài toán IMO Shortlist 2009, Problem G8
[Only registered and activated users can see links. ]
[RIGHT][I][B]Nguồn: MathScope.ORG[/B][/I][/RIGHT]
 
Nguyen Van Linh is offline   Trả Lời Với Trích Dẫn
The Following 5 Users Say Thank You to Nguyen Van Linh For This Useful Post:
dangvip123tb (01-04-2015), huynhcongbang (27-03-2015), pco (27-03-2015), thaygiaocht (27-03-2015), thiendieu96 (27-03-2015)
Old 27-03-2015, 12:49 AM   #29
Nguyen Van Linh
Moderator
 
Tham gia ngày: Aug 2009
Đến từ: Hà Nội
Bài gởi: 277
Thanks: 69
Thanked 323 Times in 145 Posts
Bài 2.
Ý a khá quen thuộc.
a) Gọi $L$ là giao của $AO$ với $(O).$ Dễ thấy $H,I,L$ thẳng hàng nên $\angle AKH=90^\circ$. Gọi $B_1, C_1$ là chân đường cao đỉnh $B,C.$
Áp dụng định lý về tâm đẳng phương cho các đường tròn $(AH), (O), (BC_1B_1C)$ suy ra $AK, B_1C_1, BC$ đồng quy tại $T.$
Ta có $\angle AID=\angle TKD.$
Mặt khác, $(TDBC)=-1$ nên theo hệ thức Maclaurin, $DI.DT=DB.DC=DK.DM$, suy ra tứ giác $TKIM$ nội tiếp, từ đó $\angle TKD=\angle TIM.$
Vậy $\angle AID=\angle MIT$, suy ra $M$ đối xứng với $N$ qua $BC$. Như vậy $N$ chuyển động trên đường tròn đối xứng với $(O)$ qua $BC.$
b) Do $(TDBC)=-1$ nên $A(TDBC)=-1$. Gọi $R$ là giao của $AH$ với $(APQ)$. Chiếu lên đường tròn $(APQ)$ suy ra $(ARPQ)=-1$ hay tứ giác $APRQ$ điều hòa, tức là $AR$ là đường đối trung của tam giác $APQ$, mà $AR$ và $AO$ đẳng giác trong $\angle BAC$ nên $AO$ là trung tuyến của $APQ$. Vậy $AJ$ luôn đi qua $O$ cố định.

Câu b thừa dữ kiện đi qua $N$. Chỉ cần đường tròn $(APQ)$ tiếp xúc với $AK$ là đủ. Hình như gần đây có mấy bài thi QG bị thừa dữ kiện thì phải.
[RIGHT][I][B]Nguồn: MathScope.ORG[/B][/I][/RIGHT]
 
Hình Kèm Theo
Kiểu File : png P2.png (35.1 KB, 39 lần tải)

thay đổi nội dung bởi: Nguyen Van Linh, 27-03-2015 lúc 12:56 AM
Nguyen Van Linh is offline   Trả Lời Với Trích Dẫn
The Following 5 Users Say Thank You to Nguyen Van Linh For This Useful Post:
dangvip123tb (01-04-2015), huynhcongbang (27-03-2015), pco (27-03-2015), thaygiaocht (27-03-2015), thiendieu96 (28-03-2015)
Old 27-03-2015, 02:00 PM   #30
huynhcongbang
Administrator

 
huynhcongbang's Avatar
 
Tham gia ngày: Feb 2009
Đến từ: Ho Chi Minh City
Bài gởi: 2,413
Thanks: 2,165
Thanked 4,188 Times in 1,381 Posts
Gửi tin nhắn qua Yahoo chát tới huynhcongbang
Mình xin gửi file latex mình đã gõ lại đề thi, đã chỉnh sửa các lỗi thiếu sót hôm qua. File lời giải chi tiết và bình luận đợt này sẽ vẫn được thực hiện như 2 năm nay.

Ngoài ra, dưới đây là bình luận của GS. Nguyễn Hùng Sơn, mọi người tham khảo thử nhé:

Theo đánh giá của tôi về độ khó thì đề năm nay có:
* 2 bài trung bình (bài 1 và bài 4) ( 50%-60% số HS sẽ làm được)
* 2,5 bài hơi khó (bài 2, bài 5a và bài 6) ( 30%-40% sẽ làm được)
* 1,5 bài khó (bài 3 và bài 5b) (khoảng 10% HS sẽ có điểm >0)
-------------------------
Sau đay tôi có 1 số nhận xét cụ thể hơn cho từng bài. Dự đoán của tôi là mức điểm để được chọn vào đôi tuyển Quốc gia có lẽ khoảng từ 23 đến 26, có nghĩa là nếu em nào làm được trọn vẹn 4 bài thì có lẽ là sẽ được chọn.
---------------
HÌNH HỌC:
---------------
Tôi rất thích bài hình học số 2. Đây là bài không dễ lắm nhưng hoàn toàn không phải sử dụng các ĐL mà chỉ các học sinh chuyên toán mới giải được.
Bài 5 là 1 bài hình học đòi hỏi khả năng vẽ hình chính xác và gọn gàng trong thời gian ngắn. Ngoài ra đề bài có lỗi, vì vậy tôi cho là sẽ không có nhiều học sinh hoàn thành tốt bài này. Tôi cho rằng đây là bài xấu nhất trong đề thi năm nay. Ban giám khảo sẽ rất đau đầu để chấm các lời giải của bài này.
---------------
ĐẠI SỐ
---------------
Cả 2 bài đại số năm nay đều không thuộc loại khó.
Bài số 6 phức tạp hơn vì phải xét rất nhiều trường hợp. Nếu thí sinh không nản và không "sợ" xét các trường hợp thì sẽ không có vấn đề với bài này.
--------------
TỔ HỢP
--------------
Bài 4 lại là 1 bài rất giống bài tổ hợp ở kì thi VMO 2015 và có lẽ cả 2 bài là của cùng 1 tác giả.
Tôi cho rằng đây là 1 bài mà nhờ nó phần lớn các em sẽ ra về với số điểm dương.
-------------
SỐ HỌC
-------------
Thực chất bài này có thể sử dụng 1 số tính chất của lý thuyết số thường dùng cho mật mã như khái niệm: multiplicative order (hoặc modulo order) của 1 số nguyên. Nhưng đây là 1 lý thuyết rất cao và có lẽ chỉ vài em dạng "đặc biệt" mới biết dùng đến. Theo tôi số học sinh làm được bài này chắc có thể đếm được trên một bàn tay.
[RIGHT][I][B]Nguồn: MathScope.ORG[/B][/I][/RIGHT]
 
File Kèm Theo
Kiểu File : pdf VN TST 2015.pdf (125.3 KB, 384 lần tải)
Kiểu File : rar VN TST 2015.rar (2.2 KB, 97 lần tải)
__________________
Sự im lặng của bầy mèo
huynhcongbang is offline   Trả Lời Với Trích Dẫn
The Following 5 Users Say Thank You to huynhcongbang For This Useful Post:
dangvip123tb (01-04-2015), son235 (30-03-2015), thaygiaocht (27-03-2015), thiendieu96 (28-03-2015), vantienducdh (27-03-2015)
Trả lời Gởi Ðề Tài Mới

Bookmarks

Ðiều Chỉnh
Xếp Bài

Quuyền Hạn Của Bạn
You may not post new threads
You may not post replies
You may not post attachments
You may not edit your posts

BB code is Mở
Smilies đang Mở
[IMG] đang Mở
HTML đang Tắt

Chuyển đến


Múi giờ GMT. Hiện tại là 05:45 PM.


Powered by: vBulletin Copyright ©2000-2024, Jelsoft Enterprises Ltd.
Inactive Reminders By mathscope.org
[page compression: 121.71 k/138.71 k (12.26%)]